You are on page 1of 13

Cambridge

LSAT

PrepTest 77 PrepTest 77
Questions 1–5

Setup:
oo six entertainers: R S T W Y Z
oo scheduled for six performances
oo each performs once
oo three AM time slots: 9 10 11
oo three PM time slots: 2 3 4

Conditions:
#1: R — Z

WY
#2:
#3: T2/T3/T4
#4: Z9/Z10/Z11

Overview:
Here’s our basic diagram:

Z T

9 10 11 2 3 4
morning afternoon

Since the first and fourth rules include Z, we can use them to generate a basic inference. Z has to be in the
morning, which in turn forces R into one of the morning slots. With R and Z occupying two of the three
morning slots, there are three ways this can go: RZ_, R_Z, or _RZ. Once R and Z are place, there will be
limited arrangements for the WY piece (second rule). Let’s make some molds to exploit these limitations.

#1A R Z W Y S/T T/S


#1B R Z S W Y T
#1C R Z S T W Y
#2A R S Z W Y T
#2B R S Z T W Y
#3A S R Z W Y T
#3B S R Z T W Y
9 10 11 2 3 4

Logic Games Solutions -1- ©Cambridge LSAT 2016


Cambridge
LSAT

PrepTest 77 PrepTest 77
Mold #1A: With R and Z in 9 and 10, and WY in 11 and 2, respectively, S and T must fill the 3 and 4
slots, in either order.
Mold #1B: With RZ in 9 and 10, and WY in 2 and 3, the third rule dictates that we place T in the 4 slot,
leaving S to fill 11.
Mold #1C: With RZ in 9 and 10, and WY in 3 and 4, we have to place T in 2 (third rule), leaving S to fill
the 11 slot.
Mold #2A: With R_Z spanning 9 through 11, and WY in 2 and 3, the third rule dictates that we place T
in 4, leaving S to fill the 10 slot.
Mold #2B: With R_Z spanning 9 through 11, and WY in 3 and 4, the third rule dictates that we place T
in 2, leaving S to fill the 10 slot.
Mold #3A: With RZ in 10 and 11, and WY in 2 and 3, the third rule demands that we place T in 4,
leaving S to fill the 9 slot.
Mold #3B: With RZ in 10 and 11, and WY in 3 and 4, the third rule forces us to place T in 2, leaving S to
fill the 9 slot.

Most of these are complete solutions with the exception of 1A. Thus, there are only eight acceptable
solutions for this game. We’re in great shape to approach the questions at this point, so let’s get to it.

1. (A) As usual, the game starts out with an elimination question. The first rule eliminates choice E and
the second rule eliminates choice D. The third rule eliminates answer choice C and the fourth rule
eliminates choice B. This leaves A as the correct answer.
(A) Correct
(B) breaks the fourth rule
(C) breaks the third rule
(D) breaks the second rule
(E) breaks the first rule

2. (B) This is only true of mold #1A, and the only remaining uncertainty is the placement of S and T in
the 3 and 4 slots. Naturally, the correct answer hinges on this uncertainty.
(A) R is in the 9 slot.
(B) Correct. As shown, S could be in either the 3 or the 4 slot.
(C) T must be in either slot 3 or slot 4.
(D) As shown, Y is in the 2 slot.
(E) As shown, Z is in the 10 slot.

3. (D) A quick glance at our molds reveals that the only slot S cannot occupy is 2. This corresponds to
answer choice D, which is correct.
(A) See molds 3A and 3B.
(B) See molds 2A and 2B.
(C) See molds 1B and 1C.
(D) Correct. If S were in 2, the WY piece would have to be in 3 and 4. This would break the third

Logic Games Solutions -2- ©Cambridge LSAT 2016


Cambridge
LSAT

PrepTest 77 PrepTest 77
rule, since T would be forced into one of the morning slots.
(E) See mold 1A.

4. (C) As mold 1A, shows, the earliest W can be is 11. Thus, answer choice C is correct. In terms of the
reasoning, if we were to place W any earlier than 11, the second rule would force us to bring Y into
one of the morning slots. However, since we already determined that both R and Z must both be in
morning slots (first and fourth rules), there’s no way to accommodate the WY piece entirely in the
morning slots.
(A) This would leave only one morning slot for R and Z.
(B) This would leave only one morning slot for R and Z.
(C) Correct. This is consistent with mold 1A.
(D) This is acceptable, but not the earliest.
(E) This is acceptable, but not the earliest.

5. (C) These complete and accurate list questions can be a pain in the butt, but since we have all the
solutions written out, let’s plough through the answer choices. All the choices include T, so we don’t
need to check it. W is next in line, so let’s check it. W need not be in the afternoon, as shown by mold
1A. This eliminates choices B and D. The next variable we need to check is Y. As our molds show, Y
is always in one of the afternoon slots, so choice A is out. Finally, let’s check S. S need not be in the
afternoon, as evidenced by molds 1B, 1C, 2A, and 2B. This leaves choice C as the correct answer.
(A) This choice is missing Y.
(B) This is missing Y and has W.
(C) Correct.
(D) This has W.
(E) This is incorrect because it includes S.

Questions 6–12

Setup:
oo six of eight bowls: L M N O P R S V
oo consecutive positions: 1 2 3 4 5 6
oo one bowl per position

Conditions:
#1: L ~M; M ~L (contrapositive)

OPS
#2: P
#3: R R1 or R6
#4: ~S2 and ~S4
#5: N N5

Logic Games Solutions -3- ©Cambridge LSAT 2016


Cambridge
LSAT

PrepTest 77 PrepTest 77
Overview:
With a lot of rules, there would typically be a lot of inferences. Unfortunately, they are mostly preclusions,
but we can include them in the main diagram. This is optional, as it does tend to clutter up the diagram.
The main thing to notice is that one of L and M must be out, due to the first rule. What this means is
that as soon as a problem excludes one of the other variables, both out slots are full, meaning that all the
remaining variables must be included. For example, if a problem forces us to exclude R, we'll know that the
"in" variables are L/M, N, O, P, S, and V. Note also that V isn't restricted in any way, so its assignment is
wide open. S is mentioned in the second and the fourth rules, so there's one small inference we can make.
P cannot be in 3, because that would force S into either 2 or 4, and break the fourth rule.

L/M
1 2 3 4 5 6 Out Out
~N ~R ~R ~R ~R ~N
~S ~N ~S
~N ~P ~N

6. (E) Typically, when a game includes an "out" group, we need to keep an eye on it for acceptability
questions. In this case, we just need to watch out for the first rule, but that won't require us to note
anything about the out group. The first rule eliminates choice A. The second rule eliminates choice D.
The third rule eliminates choice C. The fourth rule doesn't help us at this point, although choice C
does break it. Finally, the fifth rule eliminates choice B, leaving E as the correct answer.
(A) breaks the first rule
(B) breaks the fifth rule
(C) breaks the third rule
(D) breaks the second rule
(E) Correct.

7. (D) With N displayed, we must place it in 5 (fifth rule). P being in triggers the second rule. We
already deduced that P can't be in 3. P can't be in 6 either, since it's flanked by O and S. Thus, P must
be in 2. From our work, we know that either O or S must be in 1, so answer choice D is correct.

O/S P S/O N
1 2 3 4 5 6
(A) L need not be displayed at all.
(B) As shown, P must be in 2.
(C) R need not be displayed at all.
(D) Correct. As we deduced, either O or S must be in 1.
(E) V need not be displayed at all.

8. (C) With L in, the first rule dictates that M must be out. What can we deduce from O in 2? O is
mentioned in the second rule, so that's the logical place to look. We already inferred that P cannot be
in 3, so without any way to satisfy the second rule, P must be out in this scenario. Now, let's see what
we can infer about the remaining variables. Since 6 is already occupied by L, the third rule forces us to
place R in 1. The fifth rule stipulates that we must place N in 5. Finally, since S can't be in 4 (fourth

Logic Games Solutions -4- ©Cambridge LSAT 2016


Cambridge
LSAT

PrepTest 77 PrepTest 77
rule), we must put S in 3, leaving V to occupy 4. With a complete solution, the incorrect answers
must be false.

R O S V N L M P
1 2 3 4 5 6 Out Out
(A) As we deduced (first rule), M is out.
(B) As shown, S is in 3.
(C) Correct.
(D) V must be in 4.
(E) V must be in 4.

9. (A) With P in 4, the second rule dictates that O and S occupy 3 and 5, in either order. Since N can
only be in 5 (fifth rule), and 5 is already occupied, N must be out in this scenario. Since R is in,
it must be in either 1 or 6. The only two remaining variables to place are one of L/M and V. This
matches with answer choice A, and it is therefore correct.

R O/S P S/O L/M N


O/S P S/O R L/M N
1 2 3 4 5 6 Out Out
(A) Correct.
(B) N is listed in place of M, so this is incorrect.
(C) S must be in either 3 or 5, and this is missing M.
(D) This list is missing L, and has R which must be in either 1 or 6.
(E) This is missing L, and has S, which has to be in either 3 or 5.

10. (B) With L in, we know that M is out (first rule). From O4, it's not obvious what else must happen, so
let's head to the answers.

L O M
1 2 3 4 5 6 Out Out
(A) To test whether N can be in, we must place it in 5, due to the fifth rule. We know that P can't
be in 3 (see our initial deductions), and there would be no other way to satisfy the second rule.
Thus, P must be out. R has to be in 6 (third rule) since 1 is already occupied. Finally, S has to be
in 3 (fourth rule), leaving V to fill slot 2. This is acceptable, so the answer is incorrect.

L V S O N R M P
1 2 3 4 5 6 Out Out

Logic Games Solutions -5- ©Cambridge LSAT 2016


Cambridge
LSAT

PrepTest 77 PrepTest 77
(B) Correct. Since P can't be in 3, placing it in 5 is the only way we could satisfy the second rule.
The problem is, at least one of N and R must be in, but the third and fifth rules prevent us from
placing them in any of the open slots.

L O P S M N/R
1 2 3 4 5 6 Out Out
(C) As our diagram for choice A shows, R can be displayed.
(D) Our diagram for choice A proves this answer wrong.
(E) This answer choice is also disproven by the diagram for A.

11. (B) With no new information, and tasked with finding an inference that we didn't notice upfront,
we'll work through the choices using our previous work, and create new test diagrams as needed. To
test an answer choice, we'll simply exclude the variable and see if doing so creates a contradiction.
(A) Our work for question 9 disproves this choice.
(B) Correct. If we exclude O, the second rule dictates that we must also exclude P. However, given
that one of the out slots is occupied by L or M, we would end up with three variables out.
(C) P can be out, as evidenced by our diagram for question 10, choice A.
(D) We have a skeletal diagram for question 7 that we can use to prove this incorrect. We can just
take L and V and place them in the first open slots (4 and 6) and be done with it.

O/S P S/O L N V
1 2 3 4 5 6
(E) We can swap V and R from the previous diagram, and the result would be the same. Since the
diagram is acceptable, this answer choice is incorrect.

O/S P S/O L N R
1 2 3 4 5 6

12. (A) Since P is displayed, the third rule dictates that O and S must also be displayed. This means that
we'll have 5 of the six slots filled, with the final slot occupies by one of L, M, and V. Let's get to work.
Applying the fifth rule, we'll place N in 5. According to the third rule, R must be in either 1 or 6.
If R is in 1, the only openings for the OPS piece would be 2 through 4, but that would break the
fourth rule, so this wouldn't work. Thus, we know that R must be in 6. As we deduced at the outset,
P cannot be in 3, so we'll put it in 2. Slot 4 is fairly open, and this is a must be true question, so let's
head to the answers.

O/S P S/O N R
1 2 3 4 5 6
(A) Correct. As we deduced, R must be in 6, next to N.
(B) V need not be displayed at all.
(C) This must be false. O and R must be separated by at least two others.
(D) V need not be displayed at all.
(E) V need not be displayed at all.

Logic Games Solutions -6- ©Cambridge LSAT 2016


Cambridge
LSAT

PrepTest 77 PrepTest 77
Questions 13–17

Setup:
oo four employees: W X Y Z
oo selecting offices by random drawing
oo each employee has ranked the offices from 1 to 4

Conditions:
#1: each employee selects an available office
#2: exactly one office per employee
#3: each employee selects his or her highest-ranked unselected office

Overview:
Wow, what a doozy this game is! Where do we even begin, let alone how will we diagram it? As far as
diagramming, that will be fairly straightforward; we're just pairing each employee with one office, so we
can do that as a vertical column or as a horizontal row. Among other things, the tricky component of
this game is that the eventual assignments are controlled by an underlying factor: which office(s) have
already been selected when a specific employee takes his or her pick. This determines with 100% certainty
which office that employee will pick. To illustrate how a scenario would unfold, let's just pretend that
the employees got to pick offices in alphabetical order. J would select Y since it's his first choice. L would
select X since it's his first choice and hasn't yet been selected. P would select Z since Y was already selected.
Finally, this would leave T to select W, since X, Y, and Z have already been selected. Whew! Here's how this
looks:

Y X Y W
J L P T
1 2 3 4

From a purely mathematical standpoint, there are 24 possible orderings of employees: four possibilities
for who chooses first; three possibilities for who chooses second; two possibilities for who chooses third;
and one possibility for who chooses last. You might be tempted to plot all these possbilities out since this
game is so peculiar, but notice that only questions 13 and 17 address the selections directly. The other three
questions address the employees' rankings from a broader perspective.

Note that, although the order of selection dictates with complete certainty the actual pairings, an employee
who chooses second doesn't necessarily pick his second-ranked office, an employee who chooses third
doesn't necessarily pick his third-ranked office, and so on. What we can say for sure is that the person who
gets to choose first will pick his first-ranked office, since none of the others have been selected at that point.
What about the person who chooses second? That employee will either choose his first-ranked office or his
second-ranked office if his first-ranked office is not available. He won't choose anything below his second-
ranked office because only one of the offices will be selected by the time he makes his choice.

What about the employee who chooses third? That employee won't be able to select his first-ranked office
since X and Y are the only first-ranked offices. Thus, he will either select his second-ranked or his third-

Logic Games Solutions -7- ©Cambridge LSAT 2016


Cambridge
LSAT

PrepTest 77 PrepTest 77
ranked office. Finally, we get to the employee who gets the last pick. On the surface, it appears that he
could select something higher than his lowest-ranked office. For instance, what if there was a diversity
of last-ranked offices? If the person's lowest-ranked office was already chosen, he would have to select
something ranked higher. Let's see if that would work within the context of the game. Since L is the only
one who doesn't rank W last, let's assume he gets the third pick and selects W. That would mean X and Z
must have been selected prior to L's turn. T could pick X first, but none of the others would choose Z next
if given the second pick. Thus, if L gets the third pick, he won't select W. Note also that L couldn't choose
Y, because that would require someone to pick W before him, but the other employees will only pick W if
given the last pick. In summary, we know that the employee who gets the third pick won't pick his fourth
choice, and the person who gets the last choice will pick W, which is either the third-ranked office (L) or
the lowest-ranked office (everyone else). Now, let's head to the questions.

X/Y W
1st 1st/2nd 2nd/3rd 3rd/4th

1 2 3 4

13. (B) To match the uniqueness of the game, we are given an orientation question where the answer
choices only specify the selections themselves; we aren't given any information about the order in
which the employees get to pick. Since this information determines the pairings, we'll have to work
backwards from the choices towards the order of selection. For each choice, we'll note which ranked
office each employee selects, and deduce whether the answer is acceptable. Keep in mind that the
employee's selection may not match his position in the random drawing. For example, an employee
who selects his second-ranked office may not have gotten the second pick.
(A) 4th; 4th; 3rd; 3rd. Since none of the employees selected his top choice, we can immediately
throw this one out.
(B) Correct. Note that all the selections correctly match each employee's preferences. Here's how this
solution looks:

X Y Z W
1st 2nd 3rd 4th
L T J P
1 2 3 4
(C) 2nd; 3rd; 2nd; 2nd. None of the employees selected his top choice, so this is out.
(D) 1st; 3rd; 3rd; 3rd. According to our deductions, only two employees can select their third-ranked
office, so this won't work.
(E) 1st; 2nd; 3rd; 4th. At first, this looks like it could work. However, look at L's selection. J must
have chosen first since he got his top pick. Since J selected Y, if L got the second pick, he would
pick his top choice, which is X.

14. (C) This must be true question gives us no new information, so we'll compare the answer choices to
our upfront deductions.

Logic Games Solutions -8- ©Cambridge LSAT 2016


Cambridge
LSAT

PrepTest 77 PrepTest 77
(A) This need not be true. The first two picks can be each employee's top-ranked office.
(B) This could be false, as evidenced by this: 1st; 2nd; 2nd; 4th.
(C) Correct. The employee who gets first choice must select his top-ranked office.
(D) This need not be true; here's an acceptable ordering: 1st; 1st; 3rd; 4th.
(E) We can have the following selection order: 1st; 1st/2nd; 2nd; 4th.

15. (A) Another question with no new information! This game is insufferable. Let's head straight to the
choices.
(A) Correct. This could be true if L gets the last pick. Putting P, T, and J in the first three slots,
respectively will create a valid solution.

1st 1st 3rd 3rd


Y X Z W
P T J L
1 2 3 4
(B) The maximum number of employees who can select their lowest-ranked office is one.
(C) The maximum number of employees who can select their second-ranked office is two.
(D) The maximum number of employees who can select their third-ranked office is two. As a side
note, if this answer were correct (which it's not), choice A would also be correct, but there can
only be one correct answer! Thus, there are two reasons to throw it out.
(E) As in D, if this answer were correct (which it's not), B would also have to be correct, but there
can only be one correct answer!

16. (E) Finally, a question with some new information! Unfortunately, it doesn't allow us to fill much in.
Keep in mind that there are three employees left to place: J, L, and T. Here's what we have so far:

4th
W
P
1 2 3 4
Let's head to the answers, and see what we can find.
(A) This wouldn't work. If J gets the first pick (Y), at least one of L and T would get his first pick
(X). If L gets the first pick (X), T could select his second-ranked office (Y), but J would have to
choose his third-ranked office (Z). Finally, if T gets the first pick (X), J would choose his first
pick (Y) irrespective of L's selection.
(B) As our diagram for 15A shows, L has to get the last pick in order for two employees to select
their third-ranked offices.
(C) This is always false. There are only two different offices that employees rank highest: X and Y.
(D) In order for J to select X, either L or T would have to select Y before him, but notice that if L or
T gets to pick before J, each one would pick X, so this can't be true.

Logic Games Solutions -9- ©Cambridge LSAT 2016


Cambridge
LSAT

PrepTest 77 PrepTest 77
(E) Correct. This could happen in a couple of ways. If L gets the second pick, then T must select X
first, leaving J to select Y third. If L gets the third pick, then J will pick Y and T will pick X, in
either order.

X Z Y W Y/X X/Y Z W
T L J P or J/T T/J L P
1 2 3 4 1 2 3 4

17. (E) Another question with no new information! They're just not going to let up on us. On to the
choices we go.
(A) We don't have any reason to doubt that this can happen. All we need is for one of the others
to select Y before J's turn, so we'll go with P. Note that we need not even fill the diagram in
completely. There are no restrictions on the order of the employees, so as long as we have
satisfied the answer choice, we can move on.

Y X
P J
1 2 3 4
(B) Our work for 15A disproves this answer choice.
(C) Our work for 16E disproves this answer choice.
(D) Since X is T's top-ranked office, he will select it if he gets first pick.
(E) Correct. This is quite difficult to see, so let's go through the reasoning. For P to select X, both Y
and Z must be selected prior to his turn. We need to preserve office X for L, so Y must be the
first office selected. J is the only employee other than P who will select Y as his first pick. At this
point, each of the two remaining employees (L and T) would select X if given the second pick,
so by the time P gets his pick, X would not be available.

Questions 18–23

Setup:
oo five volunteers: H J K M N
oo three committees: X Y Z
oo each committee has three volunteers
oo three positions on each committee: L S T
oo one position per volunteer per committee

Conditions:
#1: N NL
#2: exactly one M
#3: KY and ~KZ

Logic Games Solutions -10- ©Cambridge LSAT 2016


Cambridge
LSAT

PrepTest 77 PrepTest 77
#4: JSY; ~JX; ~JZ

Overview:
Notice right away that we're assigning five elements to nine positions, so we're going to be placing some
elements more than once (although never more than once on a committee). The dynamics of the game
are such that, because each committee has three members, if two of the members can't be on a specific
committee, the remaining three must be on it. From the third and the fourth rules, we know that K and J
can't be assigned to Z. This means that we'll have to assign all three others (H, M, and N) to it. As dictated
by the first rule, N must be the leader, leaving the other two to fill the S and T positions. Now that we've
assigned an M, the second rule tells us that M is done for the game. With both J (fourth rule) and M not
assigned to X, we have to assign the other three (H, K, and N) to that committee. To satisfy the first rule,
we'll assign N to the leader position. Committee Y will get J, K (third rule), and one of H and N; M is
precluded from Y due to the second rule. At this point, we're in a great position to approach the questions.

T H/K K/ H/M
S K/H J M/H
L N /K N
X Y Z
~M

We can take things a step further and create a set of two molds.

#1 T H/K H/K H/M #2 T H/K K H/M


S K/H J M/H S K/H J M/H
L N K/H N L N N N
X Y Z X Y Z

Mold #1: If H is the third member of committee Y, we can't make any additional inferences about which
positions H and K occupy on Y.
Mold #2: If N is the third member of committee Y, the first rule comes into play, and he'll be the leader,
leaving K for the treasurer role.

18. (E) The fact that the first question only addresses one of the committees is a big clue that there were
upfront inferences to be made. Let's check the choices against our diagram. Only choice E has N as
the leader, so it's correct.
(A) This breaks the fourth rule, and it's also missing N in the leader position.
(B) J needs to be swapped out with N for this to work.
(C) K needs to be swapped out with N for this to work.
(D) The committee has the correct members, but N doesn't have the leader position, so this is out.
(E) Correct. This matches up with our inferences.

19. (C) This could be true of either mold, so we'll quickly sketch the possibilities and head to the answer

Logic Games Solutions -11- ©Cambridge LSAT 2016


Cambridge
LSAT

PrepTest 77 PrepTest 77
choices.

T K K H/M T K K H/M
S H J M/H S H J M/H
L N H N or L N N N
X Y Z X Y Z
(A) This is acceptable. Either H or N will be in this role.
(B) This is okay; note that this would be acceptable under all scenarios.
(C) Correct. As our diagrams show, H was forced into the S position of X.
(D) This is acceptable under any scenario.
(E) This choice is logically equivalent to A within the context of the question, so it's incorrect.

20. (B) Since this is a must be true except question, we're looking for the answer choice that could be, but
doesn't have to be true.
(A) This must be true, so it's out.
(B) Correct. As mold #2 shows, H need not be assigned to Y.
(C) This is true of both molds, so it's incorrect.
(D) Yes, our molds show that this must be true.
(E) As our molds show, this must be true.

21. (B) This could only be true of mold #1. We're asked to identify which committee(s) are fully
determined. Only Y is fully determined, so choice B is correct.

T H/K H H/M
S K/H J M/H
L N K N
X Y Z

22. (C) We're looking for the answer choice which locks down all the assignments. Let's check the choices
against our molds.
(A) Although this triggers mold #1, it doesn't solidify the assignments for X and Z.
(B) Although this solidifies the assignments for X and Z, it doesn't tell us which mold to use.
(C) Correct. This could only be true of mold #1, and it locks down all the assignments.

T H H H
S K J M
L N K N
X Y Z
(D) This doesn't single out one of the molds, and it doesn't determine the positions of H and M on

Logic Games Solutions -12- ©Cambridge LSAT 2016


Cambridge
LSAT

PrepTest 77 PrepTest 77
committee Z.
(E) This triggers mold #2, but it still leaves the uncertain assignments up in the air.

23. (C) At last, we reach the final question of the section, and it's of the dreaded equivalent rule type! Let's
work carefully and efficiently through the choices.
(A) This would be good if H were limited to two committees, but he can be on three (mold #1).
(B) Since J is precluded from being on X and Z (fourth rule), this would mean that M isn't assigned
to any of the three. That won't work.
(C) Correct. This fits perfectly. If K is on more committees than M, and K is limited to two (third
rule), K must be on exactly one. Don't forget that the setup conditions ensure that each
volunteer is on at least one committee.
(D) This does the opposite of what we need. Instead of limiting M to one, it actually forces us to
assign at least three Ms. It also presents an impossibility in that if H were on three committees,
M would have to be on four.
(E) This would work if N were limited to two committees, but he can be on three.

Logic Games Solutions -13- ©Cambridge LSAT 2016

You might also like